What is a more efficient approach for finding the integral of tanhx?

  • Thread starter Thread starter azatkgz
  • Start date Start date
  • Tags Tags
    Integral
Click For Summary
The discussion focuses on the integral of tanh(x) and highlights a common mistake in substitution methods. The initial approach incorrectly retains dx instead of substituting it with du after defining u as tanh(x). A more efficient method is suggested, involving the use of cosh^2(x) and a u^2 substitution for simplification. This alternative approach aims to streamline the integration process and avoid errors. Correcting the substitution is crucial for accurately solving the integral of sqrt(tanh(x)).
azatkgz
Messages
182
Reaction score
0
What's wrong with my solution?

\int \sqrt {\tanh x}dx

for u = \tanh x\rightarrow du = \frac {dx}{\cosh^2x}


\int \sqrt {\tanh x}dx = \int\sqrt {u}\cosh^2x dx = \int\frac {\sqrt {u}dx}{1 - u} = \int \frac {du}{2(1 + \sqrt {u})} - \int \frac {du}{2(1 - \sqrt {u})}

for v = (1 + \sqrt {u})\rightarrow dv = \frac {du}{2\sqrt {u}}and for z = (1 - \sqrt {u})\rightarrow dz = - \frac {du}{2\sqrt {u}}


\int\frac {dv(v - 1)}{v} + \int\frac {dz(1 - z)}{z} = v - \ln v + \ln z - z

\int \sqrt {\tanh x}dx = \ln (\frac {1 - \sqrt {\tanh x}}{1 + \sqrt {\tanh x}}) + 2\sqrt {\tanh x}
 
Physics news on Phys.org
I=\int \sqrt{tanhx}dx

u=tanhx

dx=cosh^2xdu

I=\int \sqrt{u}cosh^2xdu=\int \frac{\sqrt{u}du}{1-u^2}

Notice, that cosh^2x=\frac{1}{1-tanh^2x}

Then, let t=\sqrt{u} \frac{dt}{du}=\frac{1}{2\sqrt{u}}=\frac{1}{2t}.

I=2\int \frac{t^2dt}{1-t^4}=\int \frac{dt}{1-t^2} + \int \frac{dt}{1+t^2}

So, your problem is: when you substitute dx you write the same dx in integral. But it is wrong! You must write du :)

Also look at cosh^2x=\frac{1}{1-tanh^2x}
 
Last edited:
it would be faster to make a u^2 substitution
 
Question: A clock's minute hand has length 4 and its hour hand has length 3. What is the distance between the tips at the moment when it is increasing most rapidly?(Putnam Exam Question) Answer: Making assumption that both the hands moves at constant angular velocities, the answer is ## \sqrt{7} .## But don't you think this assumption is somewhat doubtful and wrong?

Similar threads

  • · Replies 105 ·
4
Replies
105
Views
6K
  • · Replies 22 ·
Replies
22
Views
3K
  • · Replies 12 ·
Replies
12
Views
3K
  • · Replies 8 ·
Replies
8
Views
2K
Replies
3
Views
1K
  • · Replies 5 ·
Replies
5
Views
2K
  • · Replies 21 ·
Replies
21
Views
2K
  • · Replies 9 ·
Replies
9
Views
2K
  • · Replies 1 ·
Replies
1
Views
1K
  • · Replies 4 ·
Replies
4
Views
2K